0% found this document useful (0 votes)
371 views25 pages

C2 - Problem Examples

Chapter - 2 Moments, Couples, Forces, Equivalent Systems Problems with solution

Uploaded by

maran.sugu
Copyright
© © All Rights Reserved
Available Formats
Download as PDF, TXT or read online on Scribd
Download as pdf or txt
0% found this document useful (0 votes)
371 views25 pages

C2 - Problem Examples

Chapter - 2 Moments, Couples, Forces, Equivalent Systems Problems with solution

Uploaded by

maran.sugu
Copyright
© © All Rights Reserved
Available Formats
Download as PDF, TXT or read online on Scribd
Download as pdf or txt
Download as pdf or txt
You are on page 1/ 25

Problem 4-99

Replace the force at A by an equivalent force and couple moment at point P.


Given:
F  375N
a  2m
b  4m
c  2m
d  1m
T  30deg
Solution :
F

375 N

MP  F cos ( T ) ( a  c)  F sin ( T ) ( b  d)
MP

737 N m

Problem 4-102
Replace the force system by an equivalent force and couple moment at point O.
Units Used :
3

kip  10 lb
Given :
F1  430lb

F2  260lb

a  2ft

e  5ft

b  8ft

f  12

c  3ft

g 5

d a

T  60deg

Solution:

 F sin ( T )
1
2
2
g

f

FRx

272.39lb

 F cos ( T )
1
2
2
g f

FRy

25 lb

FR

274 lb

T1

5.24 deg

FRx  F 2

FRy  F 2

FR 

FRx  FRy

FRy

T 1  atan

FRx

Mo  F 1 cos ( T ) ( a)  F1 sin ( T ) ( b)  F2

f
2

g f

Mo

4.61 kip ft

Problem 4-125
Replace the force and couple-moment system by an equivalent resultant force and couple
moment at point O. Express the results in Cartesian vector form.
Units Used:
kN  1000N
Given:
M = Mx i  My j  Mz k
F = F x i  Fy j  Fz k
Mx  20kN m

Fx  8kN

My  70kN m

Fy  6kN

Mz  20kN m

Fz  8kN

a  3m
b  3m

e  5m

c  4m

f  6m

d  6m

g  5m

Solution:

Fx

F  Fy

Fz

FR  F

Mx

M  My

Mz

MR  M  r u F

f
r e
g

FR

8
6 kN

MR

10
18 kN m

56

Problem 4-127
Replace the force and couple-moment system by an equivalent resultant force and couple
moment at point Q. Express the results in Cartesian vector form.
Units Used:
kN  1000N
Given:
M = Mx i  My j  Mz k
F = Fx i  Fy j  Fz k
Mx  20kN m

Fx  8kN

My  70kN m

Fy  6kN

Mz  20kN m

Fz  8kN

a  3m
b  3m

e  5m

c  4m

f  6m

d  6m

g  5m

Solution:

Fx

F  Fy
F
z

FR  F

Mx

M  My
M
z

MR  M  r u F

0
e
r

FR

8
6 kN

MR

10
30 kN m

20

Problem 4-161
Determine the coordinate direction angles of F, which is applied to the end A of the pipe
assembly, so that the moment of F about O is zero.

Units Used:
Given:

F  20lb
a  8in
b  6in

c  6in
d  10in

Solution:

Require Mo = 0. This happens when force F is directed either towards or away from point O.

c
r  a  b
d

u

r
r

0.3
0.8

0.5

If the force points away from O, then

D
E  acos ( u)
J

D
E
J

70.8
39.8 deg

56.7

If the force points towards O, then

D
E  acos ( u)
J

D
E
J

109.2
140.2 deg

123.3

Problem 4-162
Determine the moment of the force F about point O. The force has coordinate direction angles
D, E, J. Express the result as a Cartesian vector.

Given:

F  20lb

a  8in

D  60deg

b  6in

E  120deg

c  6in

J  45deg

d  10in

Solution:

c
r  a  b
d

cos ( D )
Fv  F cos ( E )
cos ( J )

M  r u Fv

298.0
15.1 lb in

200.0

Problem 4-166
Determine the resultant couple moment of the two couples that act on the assembly. Member
OB lies in the x-z plane.

Given :
F1  400N
F2  150N
a  500mm
b  600mm
c  400mm
T  45deg

Solution:

F1
F1v  0
0

0
F2v  F2
0

b cos T
r1B 
0

b sin T

0
r1A  a

M  r1B u F1v  r1A u F1v  r2 u F2v

b cos T
c
r2 

b sin T
63.64

M 169.71
Nm
263.64

Problem 4-167
Replace the force F having acting at point A by an equivalent force and couple moment at point
C.
Given:

F  50lb
a  10ft
b  20ft
c  15ft
d  10ft
e  30ft
Solution:

d
rAB  c
e

Fv  F

rAB
rAB

0
rCA  a  b

e
FR  Fv

MR  rCA u F v

FR

14.3
21.4 lb

42.9

MR

1928.6
428.6 lb ft

428.6

Problem 4-169
The horizontal force F acts on the handle of the wrench. Determine the moment of this force
about point O. Specify the coordinate direction angles D, E, J of the moment axis.
Given:

F  30N
a  50mm
b  200mm
c  10mm
T  45deg
Solution :

sin ( T )
Fv  F cos ( T )

MO  rOA u F v

D
MO
E  acos
J
MO

c
rOA  b
a

MO

D
E
J

1.06
1.06 N m

4.03
75.7
75.7 deg

159.6

Problem 4-137
Replace the three forces acting on the plate by a wrench. Specify the magnitude of the force
and couple moment for the wrench and the point P(x, y) where its line of action intersects the
plate.
Units Used:

kN  10 N

Given :
FA  500N

a  4m

FB  800N

b  6m

FC  300N
Solution:

FA

FR  FC

FB
Guesses

x  1m

y  1m

M  100N m

Given

M
 y u FR =

FR
0
FR

M
x  Find ( M  x  y)
y

b 0 0 0
a u F  a u 0
C

0 0 0 FB

3.07 kN m

1.163 m

2.061

Problem 4-138
Replace the three forces acting on the plate by a wrench. Specify the magnitude of the force and
couple moment for the wrench and the point P(y,z) where its line of action intersects the plate.

Units Used :
Given :
FA  80lb

a  12ft

FB  60lb

b  12ft

FC  40lb
Solution:

FC

FR  FB

FA
Guesses

Given

y  1ft

z  1ft

M
 y u FR =

FR
z
FR

M
y  Find ( M  y  z)
z

M  1lb ft

0 FC 0 0
a u

0  a u FB

0 0 b 0

624 lb ft

0.414 ft

8.69

Sample Problem 2/1

y
F1 = 600 N

F2 = 500 N

The forces F1, F2, and F3, all of which act on point A of the bracket, are
specied in three different ways. Determine the x and y scalar components of
each of the three forces.

35

0.1 m
x

Solution.

The scalar components of F1, from Fig. a, are

0.2 m

F1x 600 cos 35 491 N

Ans.

F1y 600 sin 35 344 N

Ans.

0.3 m
F3 = 800 N
B

The scalar components of F2, from Fig. b, are

0.4 m

F2x 500(45) 400 N

Ans.

3
F2y 500(5) 300 N

Ans.

F1 x

Ans.
Ans.

Alternatively, the scalar components of F3 can be obtained by writing F3 as


a magnitude times a unit vector nAB in the direction of the line segment AB.
Thus,
l

F3 F3nAB F3

F3 y
0.2 m

4
A

F2 x

0.4 m

(c)

(b)

Helpful Hints

You should carefully examine the


geometry of each component determination problem and not rely on
the blind use of such formulas as
Fx F cos and Fy F sin .

0.2i 0.4j
AB
800
AB
(0.2)2 (0.4)2
800 [0.447i 0.894j]
358i 716j N

A unit vector can be formed by di-

The required scalar components are then


F3x 358 N

Ans.

F3y 716 N

Ans.

which agree with our previous results.

00

F2 y

Then F3x F3 sin 800 sin 26.6 358 N

(a)

F2 = 500 N

F3y F3 cos 800 cos 26.6 716 N

35
A

=8

26.6
0.2
0.4

F3 x

A
F3

Note that the angle which orients F2 to the x-axis is never calculated. The cosine
and sine of the angle are available by inspection of the 3-4-5 triangle. Also note
that the x scalar component of F2 is negative by inspection.
The scalar components of F3 can be obtained by rst computing the angle
of Fig. c.
tan1

F1 = 600 N

F1 y

viding any vector, such as the geol


metric position vector AB , by its
length or magnitude. Here we use
the overarrow to denote the vector
which runs from A to B and the
overbar to determine the distance
between A and B.

Sample Problem 2/2

P = 800 lb

Combine the two forces P and T, which act on the xed structure at B, into
a single equivalent force R.

lb

60

Graphical solution.

The parallelogram for the vector addition of forces T and

P is constructed as shown in Fig. a. The scale used here is 1 in. 800 lb; a scale

C 60

of 1 in. 200 lb would be more suitable for regular-size paper and would give
greater accuracy. Note that the angle a must be determined prior to construction
of the parallelogram. From the given gure
tan

6 sin 60
BD

0.866
3

6 cos 60
AD

49

00

lb

R
(a)

Helpful Hints

Note the repositioning of P to permit parallelogram addition at B.

R2 (600)2 (800)2 2(600)(800) cos 40.9 274,300


R 524 lb

Ans.

The triangle for the vector addition of T and P is shown

Geometric solution.

800 lb

in Fig. b. The angle is calculated as above. The law of cosines gives

40.9

Measurement of the length R and direction of the resultant force R yields the
approximate results
R 525 lb

Ans.
B

From the law of sines, we may determine the angle which orients R. Thus,
524
600

sin sin 40.9

800 lb

600 lb

sin 0.750

48.6

Ans.
T

(b)

Algebraic solution.

By using the x-y coordinate system on the given gure,

we may write

Note the repositioning of F so as to


preserve the correct line of action of
the resultant R.

Rx Fx 800 600 cos 40.9 346 lb


Ry Fy 600 sin 40.9 393 lb
The magnitude and dipection of the resultant force R as shown in Fig. c are then
R Rx Ry
2

tan1

Ry
Rx

(346)2

tan

(393)2

524 lb

393
48.6
346

Ans.
Ans.

Rx = 346 lb

Ry = 393 lb

The resultant R may also be written in vector notation as


R Rxi Ry j 346i 393j lb

Ans.

(c)

Sample Problem 2/3

y
j

The 500-N force F is applied to the vertical pole as shown. (1) Write F in
terms of the unit vectors i and j and identify both its vector and scalar components. (2) Determine the scalar components of the force vector F along the
x- and y-axes. (3) Determine the scalar components of F along the x- and y-axes.

y
30
x

j
A
i

Solution.

Part (1).

From Fig. a we may write F as

F = 500 N

30

F (F cos )i (F sin )j

(500 cos 60)i (500 sin 60)j


(250i 433j) N

x
y

Ans.

Fx

The scalar components are Fx 250 N and Fy 433 N. The vector components are Fx 250i N and Fy 433j N.

Part (2). From Fig. b we may write F as F 500i N, so that the required
scalar components are
Fx 500 N

Fy 0

Fy
sin 60

500
sin 30

F
F
i

Ans.
(a)

(b)
y

30

F = 500 N

Fy 866 N

(c)

Helpful Hint
Fy 866 N

Ans.

Obtain Fx and Fy graphically and


compare your results with the calculated values.
a
F1 = 100 N

Forces F1 and F2 act on the bracket as shown. Determine the projection Fb


of their resultant R onto the b-axis.

30
20
b

F2 = 80 N

Solution. The parallelogram addition of F1 and F2 is shown in the gure.


Using the law of cosines gives us

(80)2

(100)2

30

90

60

Sample Problem 2/4

R2

Fx

60

90

Fy

Fx 1000 N

The required scalar components are then


Fx 1000 N

= 60

Fy

Part (3). The components of F in the x- and y-directions are nonrectangular and are obtained by completing the parallelogram as shown in Fig. c. The
magnitudes of the components may be calculated by the law of sines. Thus,
Fx
500

sin 90 sin 30

2(80)(100) cos 130

R 163.4 N

0N

The gure also shows the orthogonal projection Fb of R onto the b-axis. Its
length is
Fb 80 100 cos 50 144.3 N

F1

Ans.

Note that the components of a vector are in general not equal to the projections of the vector onto the same axes. If the a-axis had been perpendicular to
the b-axis, then the projections and components of R would have been equal.

10

50

80 N
F2

50

Fb
b

Sample Problem 2/5

2m
A

Calculate the magnitude of the moment about the base point O of the 600-N
force in ve different ways.

40
600 N

4m

Solution.

(I)

The moment arm to the 600-N force is


O

d 4 cos 40 2 sin 40 4.35 m


2m

By M Fd the moment is clockwise and has the magnitude


MO 600(4.35) 2610 N  m
(II)

40

Ans.

4m

600 N
40 d

Replace the force by its rectangular components at A


F1 600 cos 40 460 N,

F2 600 sin 40 386 N

MO 460(4) 386(2) 2610 N  m

F1 = 600 cos 40

2m

By Varignons theorem, the moment becomes


Ans.
4m

F2 = 600 sin 40

(III) By the principle of transmissibility, move the 600-N force along its
line of action to point B, which eliminates the moment of the component F2. The
moment arm of F1 becomes
O

d1 4 2 tan 40 5.68 m

and the moment is

F2

MO 460(5.68) 2610 N  m

F1
A

Ans.

(IV) Moving the force to point C eliminates the moment of the component
F1. The moment arm of F2 becomes
d2 2 4 cot 40 6.77 m

d1

C
O

d2

and the moment is

F2

MO 386(6.77) 2610 N  m

Ans.

(V) By the vector expression for a moment, and by using the coordinate
system indicated on the gure together with the procedures for evaluating cross
products, we have

The required geometry here and in


similar problems should not cause difculty if the sketch is carefully drawn.

This procedure is frequently the

MO r F (2i 4j) 600(i cos 40 j sin 40)

shortest approach.

The fact that points B and C are not

2610k N  m
The minus sign indicates that the vector is in the negative z-direction. The magnitude of the vector expression is
MO 2610 N  m

Helpful Hints

Ans.

on the body proper should not cause


concern, as the mathematical calculation of the moment of a force does not
require that the force be on the body.

Alternative choices for the position

vector r are r d1 j 5.68j m and


r d2i 6.77i m.

F1

Sample Problem 2/6

The trap door OA is raised by the cable AB, which passes over the small frictionless guide pulleys at B. The tension everywhere in the ca`le is T, and this tension applied at A causes a moment MO about the hinge at O. Plot the quantity MO/T
as a function of the door elevation angle over the range 0 90 and note minimum and maximum values. What is the physical signicance of this ratio?

0.5 m
O

We begin by constructing a gure which shows the tension force T


acting directly on the door, which is shown in an arbitrary angular position . It
should be clear that the direction of T will vary as varies. In order to deal with
this variation, we write a unit vector nAB which aims T:

Solution.

rAB
T

rOB

Using the x-y coordinates of our gure, we can write

rAB rOB rOA


nAB r
rAB
AB

0.3 m
A

0.4 m

rOA

rOB 0.4j m and rOA 0.5(cos i sin j) m


Helpful Hints

So
rAB rOB rOA 0.4j (0.5)(cos i sin j)

Recall that any unit vector can be

0.5 cos i (0.4 0.5 sin )j m

written as a vector divided by its


magnitude. In this case the vector in
the numerator is a position vector.

and
rAB (0.5 cos )2 (0.4 0.5 sin )2

0.5

0.41 0.4 sin m

0.4

The desired unit vector is


rAB 0.5 cos i (0.4 0.5 sin )j
nAB r
AB
0.41 0.4 sin

MO
,
T

Our tension vector can now be written as


i (0.4 0.5 sin )j
0.5 cos0.41

0.4 sin

i (0.4 0.5 sin )j


0.5 cos0.41

0.4 sin

0.2T cos
0.41 0.4 sin

In the expression M r F, the po-

sition vector r runs from the moment center to any point on the line
of action of F. Here, rOB is more convenient than rOA.

0.2T cos
0.41 0.4 sin

and the requested ratio is


MO
0.2 cos

T
0.41 0.4 sin

Recall that any vector may be written as a magnitude times an aiming unit vector.

The magnitude of MO is
MO

0 10 20 30 40 50 60 70 80 90
, deg

The moment of T about point O, as a vector, is MO rOB T, where rOB 0.4j m, or

0.2
0.1

T TnAB T

MO 0.4j T

0.3
m

Ans.

which is plotted in the accompanying graph. The expression MO/T is the moment
arm d (in meters) which runs from O to the line of action of T. It has a maximum
value of 0.4 m at 53.1 (at which point T is horizontal) and a minimum value of
0 at 90 (at which point T is vertical). The expression is valid even if T varies.
This sample problem treats moments in two-dimensional force systems, and
it also points out the advantages of carrying out a solution for an arbitrary position, so that behavior over a range of positions can be examined.

Sample Problem 2/7


M

The rigid structural member is subjected to a couple consisting of the two


100-N forces. Replace this couple by an equivalent couple consisting of the two
forces P and P, each of which has a magnitude of 400 N. Determine the proper
angle .

40
P

100

Solution.

The original couple is counterclockwise when the plane of the forces


is viewed from above, and its magnitude is
100

[M Fd]

M 100(0.1) 10 N  m

The forces P and P produce a counterclockwise couple


60

100

M 400(0.040) cos

100 N

Equating the two expressions gives

100 N
Dimensions in millimeters

10 (400)(0.040) cos
10

cos1 16 51.3

P = 400 N

Ans.

Helpful Hint

40 mm

Since the two equal couples are parallel free vectors, the only dimensions

which are relevant are those which give the perpendicular distances between
the forces of the couples.

P = 400 N

Sample Problem 2/8

80 lb

Replace the horizontal 80-lb force acting on the lever by an equivalent system consisting of a force at O and a couple.

We apply two equal and opposite 80-lb forces at O and identify the
counterclockwise couple

[M Fd]

M 80(9 sin 60) 624 lb-in.

60

Solution.

Ans.

80 lb

80 lb

Thus, the original force is equivalent to the 80-lb force at O and the 624-lb-in.
couple as shown in the third of the three equivalent gures.

Helpful Hint

The reverse of this problem is often encountered, namely, the replacement


of a force and a couple by a single force. Proceeding in reverse is the same as
replacing the couple by two forces, one of which is equal and opposite to the
80-lb force at O. The moment arm to the second force would be M/F
624/80 7.79 in., which is 9 sin 60, thus determining the line of action of
the single resultant force of 80 lb.

80 lb 80 lb

O
80 lb
624 lb-in.

Sample Problem 2/9

Determine the resultant of the four forces and one couple which act on the
plate shown.

2m

5m

60 N
50 N
45

Solution.

Point O is selected as a convenient reference point for the forcecouple


system which is to represent the given system.

2m

[Rx Fx]

Rx 40 80 cos 30 60 cos 45 66.9 N

2m

[Ry Fy]

Ry 50 80 sin 30 60 cos 45 132.4 N

140 Nm
80 N

[R Rx2 Ry2]

tan

Ry

Ans.

132.4
63.2
66.9

Ans.

tan

Rx

[MO (Fd)]

40 N
1m

R (66.9)2 (132.4)2 148.3 N


1

y
R = 148.3 N

MO 140 50(5) 60 cos 45(4) 60 sin 45(7)

(a)

237 N  m

= 63.2

|MO| =
237 Nm

148.3d 237

d 1.600 m

Ry b MO

and

(b)

r R MO
where r xi yj is a position vector running from point O to any point on the
line of action of R. Substituting the vector expressions for r, R, and MO and carrying out the cross product result in
(xi yj) (66.9i 132.4j) 237k
(132.4x 66.9y)k 237k
Thus, the desired line of action, Fig. c, is given by
132.4x 66.9y 237

By setting y 0, we obtain x 1.792 m, which agrees with our earlier calcula-

63.2

1.600 m
A
O

(c)

132.4 x 66.9 y =
237

O
R

x
B

237
1.792 m
b
132.4

Alternatively, the y-intercept could have been obtained by noting that the moment about O would be due to Rx only.
A more formal approach in determining the nal line of action of R is to use
the vector expression

tion of the distance b.

R = 148.3 N

Ans.

Hence, the resultant R may be applied at any point on the line which makes a
63.2 angle with the x-axis and is tangent at point A to a circle of 1.600-m radius
with center O, as shown in part b of the gure. We apply the equation Rd MO in
an absolute-value sense (ignoring any sign of MO) and let the physics of the situation, as depicted in Fig. a, dictate the nal placement of R. Had MO been counterclockwise, the correct line of action of R would have been the tangent at point B.
The resultant R may also be located by determining its intercept distance b
to point C on the x-axis, Fig. c. With Rx and Ry acting through point C, only Ry
exerts a moment about O so that

The forcecouple system consisting of R and MO is shown in Fig. a.


We now determine the nal line of action of R such that R alone represents
the original system.
[Rd MO]

30

Helpful Hints

We note that the choice of point O as


a moment center eliminates any moments due to the two forces which
pass through O. Had the clockwise
sign convention been adopted, MO
would have been 237 N  m, with
the plus sign indicating a sense
which agrees with the sign convention. Either sign convention, of
course, leads to the conclusion of a
clockwise moment MO.

Note that the vector approach


yields sign information automatically, whereas the scalar approach
is more physically oriented. You
should master both methods.

Sample Problem 2/10

F = 100 N
z

A force F with a magnitude of 100 N is applied at the origin O of the axes


x-y-z as shown. The line of action of F passes through a point A whose coordinates are 3 m, 4 m, and 5 m. Determine (a) the x, y, and z scalar components of
F, (b) the projection Fxy of F on the x-y plane, and (c) the projection FOB of F
along the line OB.

4m
A

5m
2m
6m

Part (a). We begin by writing the force vector F as its magnitude


F times a unit vector nOA.

Solution.

F FnOA F

3i 4j 5k

3m

6m

OA
100
OA
32 42 52

x
z

100[0.424i 0.566j 0.707k]

42.4i 56.6j 70.7k N


The desired scalar components are thus

Fx 42.4 N
Part (b).

Fz

Fy 56.6 N

Fz 70.7 N

The cosine of the angle xy between F and the x-y plane is


cos xy

32 42
32 42 52

Fy

xy

0.707

so that Fxy F cos xy 100(0.707) 70.7 N


Part (c).

Ans.

Fxy = 70.7 N

Ans.

Fx

The unit vector nOB along OB is

nOB

6i 6j 2k
OB

0.688i 0.688j 0.229k


OB
62 62 22

The scalar projection of F on OB is

FOB F  nOB (42.4i 56.6h 70.7k)  (0.688i 0.688j 0.229k)

(42.4)(0.688) (56.6)(0.688) (70.7)(0.229)


84.4N

nOB

Ans.

y
FOB = 84.4 N

If we wish to express the projection as a vector, we write


FOB F  nOBnOB

O
x

84.4(0.688i 0.688j 0.229k)


58.1i 58.1j 19.35k N

Helpful Hints

In this example all scalar components


are positive. Be prepared for the case
where a direction cosine, and hence
the scalar component, are negative.

The dot product automatically nds


the projection or scalar component
of F along line OB as shown.

Sample Problem 2/11

Determine the moment of force F about point O (a) by inspection and (b) by
the formal cross-product denition MO r F.

a
F

Solution. (a) Because F is parallel to the y-axis, F has no moment about that
axis. It should be clear that the moment arm from the x-axis to the line of action
of F is c and that the moment of F about the x-axis is negative. Similarly, the
moment arm from the z-axis to the line of action of F is a and the moment of F
about the z-axis is positive. So we have
MO cFi aFk F(ci ak)

x
z

Ans.

(b) Formally,

MO r F (ai ck) Fj aFk cFi


F(ci ak)

F
r

Ans.

Helpful Hint

Again we stress that r runs from the moment center to the line of action of F.
Another permissible, but less convenient, position vector is r ai bj ck.

Sample Problem 2/12

1.6 m

The turnbuckle is tightened until the tension in cable AB is 2.4 kN. Determine the moment about point O of the cable force acting on point A and the
magnitude of this moment.

2m

Solution.

We begin by writing the described force as a vector.

O
y

T TnAB 2.4

0.8 1.5 2
0.8i 1.5j 2k
2

1.5 m

0.8 m

0.731i 1.371j 1.829k kN

z
1.6 m

The moment of this force about point O is

MO rOA T (1.6i 2k) (0.731i 1.371j 1.829k)


2.74i 4.39j 2.19k kN  m

rOA

Ans.

This vector has a magnitude


MO 2.742 4.392 2.192 5.62 kN  m

2m

Ans.

Helpful Hint

The student should verify by inspection the signs of the moment components.

y
x

1.5 m
B

0.8 m

Sample Problem 2/13

A tension T of magnitude 10 kN is applied to the cable attached to the top A


of the rigid mast and secured to the ground at B. Determine the moment Mz of T
about the z-axis passing through the base O.

15 m

Solution (a).

The required moment may be obtained by nding the component along the z-axis of the moment MO of T about point O. The vector MO is
normal to the plane dened by T and point O, as shown in the accompanying gure. In the use of Eq. 2/14 to nd MO, the vector r is any vector from point O to
the line of action of T. The simplest choice is the vector from O to A, which is
written as r 15j m. The vector expression for T is
T TnAB 10

O
x
z

9m

12 m
B

(12) (15) (9)


12i 15j 9k
2

Helpful Hints

We could also use the vector from O

10(0.566i 0.707j 0.424k) kN

to B for r and obtain the same result,


but using vector OA is simpler.

From Eq. 2/14,


[MO r F]

T = 10 kN

MO 15j 10(0.566i 0.707j 0.424k)

It is always helpful to accompany your

150(0.566k 0.424i) kN  m
The value Mz of the desired moment is the scalar component of MO in the
z-direction or Mz MO  k. Therefore,
Mz 150(0.566k 0.424i)  k 84.9 kN  m

Ans.

The minus sign indicates that the vector Mz is in the negative z-direction. Ex-

vector operations with a sketch of the


vectors so as to retain a clear picture
of the geometry of the problem.

Sketch the x-y view of the problem


and show d.

pressed as a vector, the moment is Mz 84.9k kN  m.

y
A

Solution (b).

The force of magnitude T is resolved into components Tz and Txy


in the x-y plane. Since Tz is parallel to the z-axis, it can exert no moment about
this axis. The moment Mz is, then, due only to Txy and is Mz Txyd, where d is
the perpendicular distance from Txy to O. The cosine of the angle between T and
Txy is 152 122 / 152 122 92 0.906, and therefore,

O
x

The moment arm d equals OA multiplied by the sine of the angle between Txy
and OA, or

z
B

12
9.37 m
122 152

Hence, the moment of T about the z-axis has the magnitude


Mz 9.06(9.37) 84.9 kN  m

Mo

Txy 10(0.906) 9.06 kN

d 15

Mz

Tz

Tx

Ans.

and is clockwise when viewed in the x-y plane.

Ty
Txy 15 m

Solution (c). The component Txy is further resolved into its components Tx and Ty.
It is clear that Ty exerts no moment about the z-axis since it passes through it, so
that the required moment is due to Tx alone. The direction cosine of T with respect
to the x-axis is 12/92 122 152 0.566 so that Tx 10(0.566) 5.66 kN. Thus,
Mz 5.66(15) 84.9 kN  m

Ans.

x
9m

12 m
B

Sample Problem 2/14

30 N
30 N

60

Determine the magnitude and direction of the couple M which will replace
the two given couples and still produce the same external effect on the block.
Specify the two forces F and F, applied in the two faces of the block parallel to
the y-z plane, which may replace the four given forces. The 30-N forces act parallel to the y-z plane.

60

100 mm

mm

60
40

mm y

25 N

50 mm

Solution. The couple due to the 30-N forces has the magnitude M1 30(0.06)
1.80 N m. The direction of M1 is normal to the plane dened by the two forces,
and the sense, shown in the gure, is established by the right-hand convention.
The couple due to the 25-N forces has the magnitude M2 25(0.10) 2.50 N  m
with the direction and sense shown in the same gure. The two couple vectors
combine to give the components

25 N
z
M2 = 2.5 Nm

My 1.80 sin 60 1.559 N  m

M
x

Mz 2.50 1.80 cos 60 1.600 N  m

Thus,

M (1.559)2 (1.600)2 2.23 N  m

y
F

Ans.

with

tan1

1.559
1 0.974 44.3
1.600 tan

Ans.

The forces F and F lie in a plane normal to the couple M, and their moment arm as seen from the right-hand gure is 100 mm. Thus, each force has the
magnitude
2.23

F 0.10 22.3 N

[M = Fd]

Ans.

60
M1 = 1.8 Nm
z

Helpful Hint

Bear in mind that the couple vectors


are free vectors and therefore have
no unique lines of action.

and the direction 44.3.

Sample Problem 2/15

A force of 40 lb is applied at A to the handle of the control lever which is attached to the xed shaft OB. In determining the effect of the force on the shaft
at a cross section such as that at O, we may replace the force by an equivalent
force at O and a couple. Describe this couple as a vector M.

40 lb
8
2
3

A
O

The couple may be expressed in vector notation as M r F,


l
where r OA 8j 5k in. and F 40i lb. Thus,

Solution.

y
x

M (8j 5k) (40i) 200j 320k lb-in.


z

Alternatively we see that moving the 40-lb force through a distance d


52 82 9.43 in. to a parallel position through O requires the addition of a
couple M whose magnitude is
M Fd 40(9.43) 377 lb-in.

tan1 8 32.0

40 lb

Ans.

The couple vector is perpendicular to the plane in which the force is shifted, and
its sense is that of the moment of the given force about O. The direction of M in
the y-z plane is given by
Ans.

(40 lb)

O
x

A
5

8
y

Sample Problem 2/16

z
700 lb-in.

Determine the resultant of the force and couple system which acts on the
rectangular solid.

50 lb

80 lb

Solution.

We choose point O as a convenient reference point for the initial


step of reducing the given forces to a forcecouple system. The resultant force is

50 lb
960
lb-in.

R F (80 80)i (100 100)j (50 50)k 0 lb

12

The sum of the moments about O is

1000 lb-in.

80 lb

MO [50(16) 700]i [80(12) 960] j [100(10) 1000]k lb-in.


100i lb-in.

16

10 100 lb
100 lb

Hence, the resultant consists of a couple, which of course may be applied at any
point on the body or the body extended.

Helpful Hints

Since the force summation is zero, we conclude that the resultant, if it exists,
must be a couple.

The moments associated with the force pairs are easily obtained by using the

M Fd rule and assigning the unit-vector direction by inspection. In many


three-dimensional problems, this may be simpler than the M r F approach.

Sample Problem 2/17

50 N

Determine the resultant of the system of parallel forces which act on the
plate. Solve with a vector approach.

0.5 m
z

Solution.

0.5 m

R F (200 500 300 50)j 350j N


MO [50(0.35) 300(0.35)]i [50(0.50) 200(0.50)]k
87.5i 125k N  m

500 N

0.35 m

200 N
300 N

The placement of R so that it alone represents the above forcecouple system is


determined by the principle of moments in vector form

z
x

r R MO

x
z

(xi yj zk) 350j 87.5i 125k

350z 87.5

Hence, x 0.357 m and z 0.250 m are the coordinates through which the
line of action of R must pass. The value of y may, of course, be any value, as
permitted by the principle of transmissibility. Thus, as expected, the variable y
drops out of the above vector analysis.

From the one vector equation we may obtain the two scalar equations
and

350xk 350zi 87.5i 125k


350x 125

0.35 m

Transfer of all forces to point O results in the forcecouple system

MO

Helpful Hint

You should also carry out a scalar


solution to this problem.

Sample Problem 2/18

x
700 N

Replace the two forces and the negative wrench by a single force R applied
at A and the corresponding couple M.

500 N
25 N m

60

40
z B

Solution.
[Rx Fx]

The resultant force has the components


Rx 500 sin 40 700 sin 60 928 N

[Ry Fy]

Ry 600 500 cos 40 cos 45 871 N

[Rz Fz]

Rz 700 cos 60 500 cos 40 sin 45 621 N

R (928)2 (871)2 (621)2 1416 N

80 mm
45
600 N
40 mm

120 mm

R 928i 871j 621k N

Thus,
and

100
mm

30
mm
A 50 mm
60 mm

Ans.

The couple to be added as a result of moving the 500-N force is


A

[M r F] M500 (0.08i 0.12j 0.05k) 500(i sin 40


j cos 40 cos 45 k cos 40 sin 45)

where r is the vector from A to B.


The term-by-term, or determinant, expansion gives

Helpful Hints

Suggestion: Check the cross-product

M500 18.95i 5.59j 16.90k N  m

The moment of the 600-N force about A is written by inspection of its x- and zcomponents, which gives

For the 600-N and 700-N forces it is

M600 (600)(0.060)i (600)(0.040)k


36.0i 24.0k N  m
The moment of the 700-N force about A is easily obtained from the moments of
the x- and z-components of the force. The result becomes

wrench points in the direction opposite to that of the 500-N force, and
we must resolve it into its x-, y-, and
z-components to be added to the
other couple-vector components.

10.5i 71.4 j 18.19k N  m

Also, the couple of the given wrench may be written


M 25.0(i sin 40 j cos 40 cos 45 k cos 40 sin 45)

Although the resultant couple vec-

16.07i 13.54 j 13.54k N  m


Therefore, the resultant couple on adding together the i-, j-, and k-terms of the
four Ms is
M 49.4i 90.5j 24.6k N  m
and

M (49.4)2 (90.5)2 (24.6)2 106.0 N  m

easier to obtain the components of


their moments about the coordinate
directions through A by inspection
of the gure than it is to set up the
cross-product relations.

The 25-N  m couple vector of the

M700 (700 cos 60)(0.030)i [(700 sin 60)(0.060)


(700 cos 60)(0.100)] j (700 sin 60)(0.030)k

results by evaluating the moments


about A of the components of the
500-N force directly from the sketch.

Ans.

tor M in the sketch of the resultants


is shown through A, we recognize
that a couple vector is a free vector
and therefore has no specied line
of action.

Sample Problem 2/19

Determine the wrench resultant of the three forces acting on the bracket.
Calculate the coordinates of the point P in the x-y plane through which the resultant force of the wrench acts. Also nd the magnitude of the couple M of the
wrench.

4
3
y

40 lb

The direction cosines of the couple M of the wrench must be the


same as those of the resultant force R, assuming that the wrench is positive. The
resultant force is

Solution.

P
40 lb
x

R (20)2 (40)2 (40)2 60 lb

R 20i 40j 40k lb

20 lb

and its direction cosines are


cos x 20/60 1/3

cos y 40/60 2/3

cos z 40/60 2/3

The moment of the wrench couple must equal the sum of the moments of
the given forces about point P through which R passes. The moments about P of
the three forces are
(M)Rx 20yk lb-in.

x=

3i

n.

P
5

y = 2 in.
M

(M)Ry 40(3)i 40xk lb-in.


(M)Rz 40(4 y)i 40(5 x)j lb-in.

Helpful Hint

We assume initially that the wrench

and the total moment is

is positive. If M turns out to be negative, then the direction of the couple vector is opposite to that of the
resultant force.

M (40 40y)i (200 40x)j (40x 20y)k lb-in.


The direction cosines of M are
cos x (40 40y)/M
cos y (200 40x)/M
cos z (40x 20y)/M
where M is the magnitude of M. Equating the direction cosines of R and M gives
40 40y

M
3

200 40x

2M
3

40x 20y

2M
3

Solution of the three equations gives


M 120 lb-in.

x 3 in.

y 2 in.

Ans.

We see that M turned out to be negative, which means that the couple vector is
pointing in the direction opposite to R, which makes the wrench negative.

You might also like